Fórum de Matemática
DÚVIDAS? Nós respondemos!

Um Fórum em Português dedicado à Matemática
Data/Hora: 27 abr 2024, 20:42

Os Horários são TMG [ DST ]




Fazer Nova Pergunta Responder a este Tópico  [ 3 mensagens ] 
Autor Mensagem
MensagemEnviado: 21 jan 2015, 03:10 
Offline

Registado: 24 jul 2014, 18:04
Mensagens: 80
Localização: Brasil
Agradeceu: 26 vezes
Foi agradecido: 1 vez(es)
calcule o volume no primeiro octante pelo cilindro x²+z²=9 e pelos planos y=x e y=3x.


Queria saber como eu transfiro para coordenadas polares. Pois não estou conseguindo achar a variação do angulo.


Resp: 18



Obrigado


Topo
 Perfil  
 
MensagemEnviado: 21 jan 2015, 03:37 
Offline

Registado: 24 nov 2014, 15:04
Mensagens: 369
Localização: Minas Gerais
Agradeceu: 45 vezes
Foi agradecido: 72 vezes
Boa noite F. Augusto!

Amigo, infelizmente não vou poder ajudá-lo mas com certeza alguém vai te ajudar por aqui :(

Também estou vendo essa matéria na faculdade mas estou bem "perdido". Deixa eu te perguntar: Essas questões que você tem enviado aqui no fórum é de qual material?

Abraço


Topo
 Perfil  
 
MensagemEnviado: 21 jan 2015, 12:21 
Offline

Registado: 17 jan 2013, 13:36
Mensagens: 2487
Localização: Lisboa
Agradeceu: 31 vezes
Foi agradecido: 1049 vezes
Bom dia,

Deve começar por esboçar a região em causa... Verá que \(\theta \in [\pi/4 , \arctan 3]\), já que olhando para o plano xy estaremos entre a recta y=x (declive =1, logo angulo = arctan 1 = pi/4) e a recta y=3x (declive =3, logo angulo = arctan 3). Quando o angulo varia nesse intervalo, a distancia à origem varia entre 0 a a distancia correspondente a um ponto sobre a recta x = 3, isto é, \(r \cos \theta =3 \Leftrightarrow r = \frac{3}{\cos \theta}\). Finalmente, z está sempre entre 0 e a altura correspondente à superfície do cilindro, isto é \(x^2+ z^2 = {9} \Leftrightarrow r^2 \cos^2 \theta + z^2 = {9} \Leftrightarrow z = \sqrt{9 - r^2\cos^2 \theta}\). FInalmente, o integral é dado por

\(\int_{\pi/4}^{\arctan 3}\left( \int_0^{3/\cos \theta} \left(\int_0^{\sqrt{9 -r^2 \cos^2\theta}} r dz\right) dr \right) d\theta = \cdots = 18\)


Topo
 Perfil  
 
Mostrar mensagens anteriores:  Ordenar por  
Fazer Nova Pergunta Responder a este Tópico  [ 3 mensagens ] 

Os Horários são TMG [ DST ]


Quem está ligado:

Utilizadores a ver este Fórum: Nenhum utilizador registado e 136 visitantes


Criar perguntas: Proibído
Responder a perguntas: Proibído
Editar Mensagens: Proibído
Apagar Mensagens: Proibído
Enviar anexos: Proibído

Pesquisar por:
Ir para: